LSAT and Law School Admissions Forum

Get expert LSAT preparation and law school admissions advice from PowerScore Test Preparation.

User avatar
 Dave Killoran
PowerScore Staff
  • PowerScore Staff
  • Posts: 5852
  • Joined: Mar 25, 2011
|
#85485
Setup and Rule Diagram Explanation

This is a Basic Linear: Balanced, Identify the Templates game.

This is a nearly ideal game to start any LSAT. The scenario assigns six racehorses to six positions, creating a Basic Linear setup:

PT28-Jun 1999 LGE-G1_srd1.png

The first rule creates a KL split-block:

PT28-Jun 1999 LGE-G1_srd2.png

Because this block rotates, no Not Laws can be drawn initially.

The second rule establishes a KN not-block:

PT28-Jun 1999 LGE-G1_srd3.png

While this block could be written out in K/N N/K not-block form, writing both prohibited blocks is easier to see visually. This explains the difference in the representations between the first rule and the second rule (not taking into account the negative in the second rule, of course). Again, no Not Laws can be drawn from this rule because it rotates.

The third rule establishes a basic sequence:

PT28-Jun 1999 LGE-G1_srd4.png

This produces two Not Laws:

PT28-Jun 1999 LGE-G1_srd5.png

The final rules places P:

PT28-Jun 1999 LGE-G1_srd6.png

Because P is assigned to position 3, the KL split-block must be assigned to positions 2 and 4, or positions 4 and 6. Thus, L and K can never be assigned to position 1, and L and K can never be assigned to position 5. Additionally, K or L must always be assigned to position 4:

PT28-Jun 1999 LGE-G1_srd7.png

In examining the diagram, two other positions are also somewhat restricted. Position 1 cannot be N, K, or L, or P (who is assigned to position 3), so position 1 must be M or O. Position 5 cannot be K, L, or P, so position 5 must be M, N, or O. Adding in that O is a random results in the final diagram:

PT28-Jun 1999 LGE-G1_srd8.png

The K and L split-block is clearly the most restricted element in the game. This restriction, in combination with the remaining rules and limitations, suggests Identifying the Templates:

PT28-Jun 1999 LGE-G1_srd9.png

If the two templates are confusing, another option would be to Identify the Possibilities:

PT28-Jun 1999 LGE-G1_srd10.png

There are seven total possibilities. By using either the Templates or Possibilities approach, the game becomes relatively easy.
You do not have the required permissions to view the files attached to this post.
 rashidzaman
  • Posts: 3
  • Joined: Aug 03, 2020
|
#78296
Why aren't K & L excluded from position 6? they need 2 more spaces not available in position 6 - just like their exclusion from position 5
User avatar
 KelseyWoods
PowerScore Staff
  • PowerScore Staff
  • Posts: 1079
  • Joined: Jun 26, 2013
|
#78373
Hi Rashid!

K and L have to have exactly one space in between them ( K/L ____ L/K ). Since P is in position 3, that means that K and L could be in 2 & 4 (with P in between them), or 4 & 6 (with O in between them).

Really there are only 7 possible arrangements of the variables in this game, which can be represented by 3 templates based on where N goes (N2, N5, N6):

  M          N          P     L/K      O       K/L
  M          K          P       L          N          O
M/O     K/L      P     L/K     O/M     N
  1          2          3          4          5          6

Hope this helps!

Best,
Kelsey
 rashidzaman
  • Posts: 3
  • Joined: Aug 03, 2020
|
#78408
Hi Kelsey
So these are templates for inclusion - I was talking about exclusion - KLN are excluded from position 1 - and KL are excluded from position 5 - shouldn't KL also be excluded from position 6?
 Jeremy Press
PowerScore Staff
  • PowerScore Staff
  • Posts: 1000
  • Joined: Jun 12, 2017
|
#78518
Hi Rashid,

K and L don't have to be excluded from position 6, because the following solutions are valid under the rules:
Screen Shot 2020-08-28 at 2.41.00 PM.png
The reason K and L have to be excluded from positions 1 and 5, though, has to do with the placement of P. If you put K in position 1, you'd have to put L in position 3 (which doesn't work, because P must be placed there). Same problem if we put L in position 1. Same problem if we put K in position 5 (L would have to go in 3, and it can't). Same problem if we put L in position 5.

Since position 4 is available to be used when one of K or L goes in position 6, K and L are allowed to go in position 6.

Let us know if that clears it up!
You do not have the required permissions to view the files attached to this post.
User avatar
 jailenea
  • Posts: 25
  • Joined: Aug 30, 2021
|
#90277
Can we please get a set up/diagrams for this game?
User avatar
 jailenea
  • Posts: 25
  • Joined: Aug 30, 2021
|
#90279
Regarding my last post, how could we diagram this with and without templates? I did templates and got 7, but that doesn't seem the best way to go in the interest of time. How could it be done otherwise?
User avatar
 Beth Hayden
PowerScore Staff
  • PowerScore Staff
  • Posts: 123
  • Joined: Sep 04, 2021
|
#90291
Hi Jailenea,

You don't necessarily have to diagram out all 7 different solutions, but it's hard to see how the K/L rule fits into the game without at least creating some rough templates, and there are so many deductions that it ends up being worth the time investment. But I'll show you how to do it the more traditional way with not laws.

As Kelsey said, there are only 3 places that N can go, so you can create three templates like she did, which fill out quite nicely.

But if you didn't see that or don't want to do full templates, you could also split up the game into two diagrams based on where K/L can go and make deductions from there. Like the other instructors said, K/L can only go in the 2/4 and 4/6 slots because of P:
Screen Shot 2021-09-10 at 1.44.07 PM.jpg
In the top scenario, N can't go in 5 because it would be next to K, and M has to come before it, so N must be in 2 and M must be in 1. Then O is leftover and has to go in 5.
Screen Shot 2021-09-10 at 2.12.43 PM.png
For the bottom scenario, M can't go in 6 (because N comes after it), so it can only be in 1 or 5. N can't go in 1 for the same reason, so it must be 5 or 6. So if you want to use not laws, you will get this:
Screen Shot 2021-09-10 at 2.18.42 PM.png
Of course, that doesn't fully capture the K/N rule or the M--N rules, which is why it helps to just split that scenario into two based on where N can go and create three templates.

Hope that's helpful!
You do not have the required permissions to view the files attached to this post.

Get the most out of your LSAT Prep Plus subscription.

Analyze and track your performance with our Testing and Analytics Package.